Difference between revisions of "2009 AMC 10B Problems/Problem 1"

m (Solution 4 (Algebra): Deleted solutions that complicate concepts.)
 
(20 intermediate revisions by the same user not shown)
Line 1: Line 1:
{{duplicate|[[2009 AMC 10B Problems|2009 AMC 10B #1]] and [[2009 AMC 12B Problems|2009 AMC 12B #1]]}}
+
{{duplicate|[[2009 AMC 10B Problems#Problem 1|2009 AMC 10B #1]] and [[2009 AMC 12B Problems#Problem 1|2009 AMC 12B #1]]}}
  
 
== Problem ==
 
== Problem ==
Each morning of her five-day workweek, Jane bought either a 50-cent muffin or a 75-cent bagel. Her total cost for the week was a whole number of dollars. How many bagels did she buy?
+
Each morning of her five-day workweek, Jane bought either a <math>50</math>-cent muffin or a <math>75</math>-cent bagel. Her total cost for the week was a whole number of dollars. How many bagels did she buy?
  
<math>\text{(A) } 1\qquad\text{(B) } 2\qquad\text{(C) } 3\qquad\text{(D) } 4\qquad\text{(E) } 5</math>
+
<math>\textbf{(A) } 1\qquad\textbf{(B) } 2\qquad\textbf{(C) } 3\qquad\textbf{(D) } 4\qquad\textbf{(E) } 5</math>
  
== Solution 1 (Algebra) ==
+
== Solution 1 (Observations: Replacements) ==
A muffin costed <math>2</math> quarters, and a bagel costed <math>3</math> quarters.  
+
If Jane bought one more bagel but one fewer muffin, then her total cost for the week would increase by <math>25</math> cents.
  
Suppose that Jane bought <math>m</math> muffins and <math>b</math> bagels, where <math>m</math> and <math>b</math> are nonnegative integers. We need:
+
If Jane bought <math>1</math> bagel, then she bought <math>4</math> muffins. Her total cost for the week would be <math>75\cdot1+50\cdot4=275</math> cents, or <math>2.75</math> dollars. Clearly, she bought one more bagel but one fewer muffin at a total cost of <math>3.00</math> dollars. Therefore, she bought <math>\boxed{\textbf{(B) } 2}</math> bagels.
 +
 
 +
~MRENTHUSIASM
 +
 
 +
== Solution 2 (Observations: Answer Choices) ==
 +
* If Jane bought <math>1</math> bagel, then she bought <math>4</math> muffins. Her total cost for the week would be <math>75\cdot1+50\cdot4=275</math> cents, or <math>2.75</math> dollars. So, <math>\textbf{(A)}</math> is incorrect.
 +
 
 +
* If Jane bought <math>2</math> bagels, then she bought <math>3</math> muffins. Her total cost for the week would be <math>75\cdot2+50\cdot3=300</math> cents, or <math>3.00</math> dollars. So, <math>\boxed{\textbf{(B) } 2}</math> is correct.
 +
 
 +
For completeness, we will check <math>\textbf{(C)},\textbf{(D)},</math> and <math>\textbf{(E)}</math> too. If you decide to use this solution on the real test, then you will not need to do that, as you want to save more time.
 +
 
 +
* If Jane bought <math>3</math> bagels, then she bought <math>2</math> muffins. Her total cost for the week would be <math>75\cdot3+50\cdot2=325</math> cents, or <math>3.25</math> dollars. So, <math>\textbf{(C)}</math> is incorrect.
 +
 
 +
* If Jane bought <math>4</math> bagels, then she bought <math>1</math> muffin. Her total cost for the week would be <math>75\cdot4+50\cdot1=350</math> cents, or <math>3.50</math> dollars. So, <math>\textbf{(D)}</math> is incorrect.
 +
 
 +
* If Jane bought <math>5</math> bagels, then she bought <math>0</math> muffins. Her total cost for the week would be <math>75\cdot5+50\cdot0=375</math> cents, or <math>3.75</math> dollars. So, <math>\textbf{(E)}</math> is incorrect.
 +
 
 +
~MRENTHUSIASM
 +
 
 +
== Solution 3 (Arithmetic) ==
 +
In this solution, all amounts are in the unit of cents.
 +
 
 +
Note that the amount spent on muffins must end in either <math>00</math> or <math>50,</math> and the amount spent on bagels must end in one of <math>00,25,50,</math> or <math>75.</math> Furthermore, the number of muffins bought and the number of bagels bought must sum to <math>5.</math>
 +
 
 +
We have two possible cases:
 
<ol style="margin-left: 1.5em;">
 
<ol style="margin-left: 1.5em;">
   <li><math>m+b=5.</math></li><p>
+
   <li>The amounts spent on muffins and bagels both end in <math>00.</math></li><p>
   <li><math>2m+3b</math> is divisible by <math>4.</math></li><p>
+
The number of muffins bought must be even, and the number of bagels bought must be a multiple of <math>4.</math> <p>
 +
In this case, there are no solutions.
 +
   <li>The amounts spent on muffins and bagels both end in <math>50.</math></li><p>
 +
The number of muffins bought must be odd, and the number of bagels bought must be <math>2</math> more than a multiple of <math>4.</math> <p>
 +
In this case, the only solution is <math>3</math> muffins and <math>2</math> bagels, from which the answer is <math>\boxed{\textbf{(B) } 2}.</math>
 
</ol>
 
</ol>
From the second condition, it is clear that <math>b</math> must be even. By a quick inspection, the only solution is <math>(m,b)=(3,2),</math> so the answer is <math>b=\boxed{\text{(B) } 2}.</math>
 
 
 
~MRENTHUSIASM
 
~MRENTHUSIASM
  

Latest revision as of 22:41, 14 September 2021

The following problem is from both the 2009 AMC 10B #1 and 2009 AMC 12B #1, so both problems redirect to this page.

Problem

Each morning of her five-day workweek, Jane bought either a $50$-cent muffin or a $75$-cent bagel. Her total cost for the week was a whole number of dollars. How many bagels did she buy?

$\textbf{(A) } 1\qquad\textbf{(B) } 2\qquad\textbf{(C) } 3\qquad\textbf{(D) } 4\qquad\textbf{(E) } 5$

Solution 1 (Observations: Replacements)

If Jane bought one more bagel but one fewer muffin, then her total cost for the week would increase by $25$ cents.

If Jane bought $1$ bagel, then she bought $4$ muffins. Her total cost for the week would be $75\cdot1+50\cdot4=275$ cents, or $2.75$ dollars. Clearly, she bought one more bagel but one fewer muffin at a total cost of $3.00$ dollars. Therefore, she bought $\boxed{\textbf{(B) } 2}$ bagels.

~MRENTHUSIASM

Solution 2 (Observations: Answer Choices)

  • If Jane bought $1$ bagel, then she bought $4$ muffins. Her total cost for the week would be $75\cdot1+50\cdot4=275$ cents, or $2.75$ dollars. So, $\textbf{(A)}$ is incorrect.
  • If Jane bought $2$ bagels, then she bought $3$ muffins. Her total cost for the week would be $75\cdot2+50\cdot3=300$ cents, or $3.00$ dollars. So, $\boxed{\textbf{(B) } 2}$ is correct.

For completeness, we will check $\textbf{(C)},\textbf{(D)},$ and $\textbf{(E)}$ too. If you decide to use this solution on the real test, then you will not need to do that, as you want to save more time.

  • If Jane bought $3$ bagels, then she bought $2$ muffins. Her total cost for the week would be $75\cdot3+50\cdot2=325$ cents, or $3.25$ dollars. So, $\textbf{(C)}$ is incorrect.
  • If Jane bought $4$ bagels, then she bought $1$ muffin. Her total cost for the week would be $75\cdot4+50\cdot1=350$ cents, or $3.50$ dollars. So, $\textbf{(D)}$ is incorrect.
  • If Jane bought $5$ bagels, then she bought $0$ muffins. Her total cost for the week would be $75\cdot5+50\cdot0=375$ cents, or $3.75$ dollars. So, $\textbf{(E)}$ is incorrect.

~MRENTHUSIASM

Solution 3 (Arithmetic)

In this solution, all amounts are in the unit of cents.

Note that the amount spent on muffins must end in either $00$ or $50,$ and the amount spent on bagels must end in one of $00,25,50,$ or $75.$ Furthermore, the number of muffins bought and the number of bagels bought must sum to $5.$

We have two possible cases:

  1. The amounts spent on muffins and bagels both end in $00.$
  2. The number of muffins bought must be even, and the number of bagels bought must be a multiple of $4.$

    In this case, there are no solutions.

  3. The amounts spent on muffins and bagels both end in $50.$
  4. The number of muffins bought must be odd, and the number of bagels bought must be $2$ more than a multiple of $4.$

    In this case, the only solution is $3$ muffins and $2$ bagels, from which the answer is $\boxed{\textbf{(B) } 2}.$

~MRENTHUSIASM

See also

2009 AMC 10B (ProblemsAnswer KeyResources)
Preceded by
First Question
Followed by
Problem 2
1 2 3 4 5 6 7 8 9 10 11 12 13 14 15 16 17 18 19 20 21 22 23 24 25
All AMC 10 Problems and Solutions
2009 AMC 12B (ProblemsAnswer KeyResources)
Preceded by
First Question
Followed by
Problem 2
1 2 3 4 5 6 7 8 9 10 11 12 13 14 15 16 17 18 19 20 21 22 23 24 25
All AMC 12 Problems and Solutions

The problems on this page are copyrighted by the Mathematical Association of America's American Mathematics Competitions. AMC logo.png